Question 8 > Find the area of the trapezoid shown below 9 19 18 21 23 I Question Help ve

Question 8 > Find The Area Of The Trapezoid Shown Below 9 19 18 21 23 I Question Help Ve

Answers

Answer 1

288 u²

1) Let's calculate the area of that trapezoid by plugging into the formula below the measures of the altitude, larger base, smaller one:

[tex]\begin{gathered} S=\frac{(B+b)h}{2} \\ S=\frac{(23+9)18}{2} \\ S=288 \end{gathered}[/tex]

2) So that trapezoid has an area of 288 u²


Related Questions

In a recent year, 26.3% of all registered doctors were female. If there were 47,400 female registered doctors that year, what was the total number of registered doctors? Round your answer to the nearest whole number.

Answers

From the problem statement we can write:

47,400 is 26.3% of total registered doctors

We need to convert this word equation to algebraic equation noting that,

• "is" means "="

,

• "of" means "x"

Also, remember to convert the percentage to decimal by dividing by 100,

[tex]\frac{26.3}{100}=0.263[/tex]

The algebraic equation, thus, is:

[tex]47,400=0.263\times\text{total}[/tex]

We let total be "t" and solve :

[tex]\begin{gathered} 47,400=0.263t \\ t=\frac{47,400}{0.263} \\ t=180228.14 \end{gathered}[/tex]

Rounding to the nearest whole number,

Total Registered Doctors = 180,228

Answer:

180,228

Let w be defined as 2 more than the number of digits in the integer w. For example, 15* = 4 (2 digits in 15 + 2). If whas 7000 digits, then what is the value of (w)*?

Answers

The number of digits in 7000 is 4

The number of digits in w=7000

[tex](w)^{\cdot}=\text{ the number of digits in w+2}[/tex]

[tex](w)^{\cdot}=\text{7000+2}[/tex][tex](w)^{\cdot}=7002[/tex]

Hence the required value is 7002.

Alec wants to purchase a new phone that costs $219.00. His current average net pay is $212.34 each week. What percent of his weekdy net pay does Alec need to save each week, for the next seven weeks, to reach
his goal? Round to the nearest hundredth (1 point)
9.69%
14.73%
O 21.76%
31.28%

Answers

Answer:

14.73%

Step-by-step explanation:

firstly let's divide the phone price into 7 equal parts. by this equation 219.00/7=31.28

So Alec needs to save $31.28 but we want the percentage.

by equation x%*212.34=31.28

x=(31.28*100)/212.34=3128/212.34=14.73

so Alec needs to save 14.73% of 212.34 each week.

Solve this system of equations by elimination. Enter your answer as an ordered pair (x,y). Do not use spaces in your answer. If your answer is no solution, type "no solution". If your answer is infinitely many solutions, type "infinitely many solutions".

Answers

5x + 2y = -12 (a)

3y + 5x =-8 (b)

First, write (b) in the ax+by=c form:

5x + 3y = -8 (b)

Now, subtract (b) to (a) to eliminate x

5x + 2y = -12

-

5x + 3y = -8

__________

-y = -4

solve for y:

Multiply both sides by -1

y=4

Replace y=4 on (a) and solve for x:

5x + 2 (4) = -12

5x + 8 = -12

5x = -12-8

5x = -20

x = -20/5

x = -4

Solution: (-4,4)

i need help with this. for 2nd option, select only one sub-option

Answers

A matrix being in row echelon form means that Gaussian elimination has operated on the rows.

A matrix is in reduced row echelon form (also called row canonical form) if it satisfies the following conditions:

- It is in row echelon form.

-The leading entry in each nonzero row is a 1 (called a leading 1).

-Each column containing a leading 1 has zeros in all its other entries.

The matrix presented on the problem satisfies all conditions, therefore, the matrix is indeed in reduced row-echelon form.

26÷2.40=10.833333 round to the nearest cent

Answers

26÷ 2.40= 10.833333

Nearest cent means ,2 numbers after decimal point

Then it is 10.83

count 2 numbers to right ,and discard rest of 3333

Then answer is = 10.83

I need an quadratic equation with -3 and 6 for this assignment

Answers

If a quadratic equation has solutions

[tex]x=a,x=b[/tex]

Then

[tex]x-a=0\text{ and x-b=0}[/tex]

Furthermore, the quadratic can be written as

[tex]\begin{gathered} y=(x-a)(x-b) \\ where,y=0 \end{gathered}[/tex]

Therefore,

[tex](x-a)(x-b)=0[/tex]

Given:

[tex]a=-3,b=6[/tex]

Hence,

[tex]\begin{gathered} (x--3)(x-6)=0 \\ (x+3)(x-6)=0 \end{gathered}[/tex]

Simplify

[tex]\begin{gathered} x(x-6)+3(x-6)=0 \\ x^2-6x+3x-18=0 \\ x^2-3x-18=0 \end{gathered}[/tex]

Hence, the quadratic equation is

[tex]x^{2}-3x-18=0[/tex]

Question 5 of 10 Solve the proportion below. 23 A 6 B. 8 C. 9 D.

Answers

[tex]\frac{x}{12.6}=\frac{45}{63}[/tex]

solve for x

[tex]\begin{gathered} 12.6\times\frac{x}{12.6}=\frac{5}{7}\times12.6 \\ x=\frac{63}{7}=9 \end{gathered}[/tex]

answer: C. 9

Solve the following system using the substitution method. Enter your answer as an ordered pair in the form (x,y). 3x-2y=55x+10y=35

Answers

System of equations

• Equation 1

[tex]3x-2y=5[/tex]

• Equation 2

[tex]5x+10y=35[/tex]

Procedure

Solving the system by substitution.

0. Isolating ,x ,from equation 2:

[tex]5x=35-10y[/tex][tex]x=\frac{35}{5}-\frac{10y}{5}[/tex][tex]x=7-2y[/tex]

2. Replacing the expression of x obtained in equation 1:

[tex]3\cdot(7-2y)-2y=5[/tex]

3. Simplifying:

[tex]21-6y-2y=5[/tex][tex]-8y=5-21[/tex][tex]y=\frac{-16}{-8}[/tex][tex]y=2[/tex]

4. Finally, we replace this value in the isolated expression of x and solve it:

[tex]x=7-2\cdot(2)[/tex][tex]x=7-4[/tex][tex]x=3[/tex]

Answer: (3, 2)

12. Jimmy is paid $14.50 per hour for a regular forty-hour work week and 5 point time and a half for any hour worked over 40. This pas week, Jimmy earned $754.00 in total pay. How many hours of overtime did Jimmy work?

Answers

Jimmy is paid $14.50 per hour for a regular forty-hour work week and 5 point time and a half for any hour worked over 40. This pas week, Jimmy earned $754.00 in total pay. How many hours of overtime did Jimmy work? ​

Let

x -----> the total hours worked

we have that

$14.50 --------> 40 hours

5.5($14.50) -------> > 40 hours

so

754=14.50*40+5.5(14.50)x

solve for x

754=580+79.75x

79.75x=754-580

79.75x=174

x=2.2 hours

What is the coordinate point location of the y-intercept of the graph below?

Answers

The y-intercept is located at the coordinate (0, 4) as shown below. Y-intercept is the point where a line or a graph crosses the y-axis.

find the value of the expression 4d ÷ c when c=3and d=6 simplify your answer

Answers

[tex]\text{ }\frac{4(6)}{3}\text{ = }\frac{24}{3}\text{ = 8}[/tex]

The linear regressionequation andcorrelation coefficientfrom the above datawas calculated to be:Predicted y = 16.2+2.45(x) with r = 0.98What is the coefficientof determination?Answer Choices:A. Coefficient of determination = 0.98B. Coefficient of determination = 0.96C. Coefficient of determination = 0.99D. Coefficient of determination cannot be determined with only the given information.

Answers

Given:

[tex]\text{ coefficient of correlation \lparen r\rparen = 0.98}[/tex]

To find:

Coefficient of determination

Explanation:

The coefficient of determination is also known as the R squared value, which is the output of the regression analysis method.

If the value of R square is zero, the dependent variable cannot be predicted from the independent variable.

So, here the required coefficient of determination is:

[tex]r^2=(0.98)^2=0.9604\approx0.96[/tex]

Final answer:

Hence, the required coefficient of determination is (B) 0.96.

Which sample size will produce the widest 95% confidence interval, given asample proportion of 0.5?A. 40B. 70C. 60D. 50

Answers

The confidence interval depends on the margin of error. When finding the margin of error, the z score corresponding to the 95% confidence level would be multiplied by the square root of the product of the estimated proportion of success and failure divided by the sample size. The greater the sample size, the smaller thie value that would be gotten from this operation. The smaller the sample size, the greater the value that would be gotten from this operation. A greater value would give a bigger margin of error. Thus, the confidence interval would be wider. Hence, the correct option for the sampe size is

A. 40

What is the most precise name for quadrilateral ABCD with vertices A(−5,7), B(6,−3), C(10,2), and D(−1,12)?A. rectangleB. parallelogramC. squareD. rhombus

Answers

Answer:

A. Rectangle

Step-by-step explanation:

Jo borrowed $3800 for 8 months from a bank at 5.5% a. how much interest did jo pay the bank for the us of it's money?b. how much did he pay total?

Answers

Let's begin by listing out the given information:

Loan (p) = $3,800

Time (t) = 8 months = 8/12 year

Interest rate (r) = 5.5%

a)

We calculate it thus:

[tex]\begin{gathered} I=\frac{p\times r\times t}{100} \\ I=\frac{3800\times5.5\times\frac{8}{12}}{100}=139.33 \\ I=\text{\$}139.33 \end{gathered}[/tex]

b)

The amount paid in total is:

[tex]\begin{gathered} A=p+I \\ A=3800+139.33=3939.33 \\ A=\text{\$}3939.33 \end{gathered}[/tex]

do you know the north Zone at the football stadium has 95 Rose there are 48 seats in a row how many people will the North end zone seat

Answers

The North zone at the football stadium has 95 rows.

There are 48 seats in a row.

How many people will the North end zone seat​?

Since there are 95 rows and each row has 48 seats, multiply them to get the total number of seats.

[tex]\begin{gathered} total\: seats=rows\times seats \\ total\: seats=95\times48 \\ total\: seats=4560 \end{gathered}[/tex]

Therefore, there are 4560 people sitting in the North zone.

which is an incorrect rounding for 53.864a) 50b) 54c) 53.9d) 53.87

Answers

Answer:

The incorrect rounding is 53.87

Explanations:

The given number is 53.864

If the number is approximated to 2 decimal places

53.864 = 53.86

If the number is approximated to 1 decimal place

53.864 = 53.9

If the number is approximated to the nearest unit

53.864 = 54

If the number is approximated to the nearest tens:

53.864 = 50

Note: 53.864 cannot be approximated to 53.87 because the third decimal place (4) is not up to 5

I need help with this statistics question please!

Answers

The margin of error of a z-confidence interval is given by: [tex]$M=z \frac{\sigma}{\sqrt{n}}$$[/tex]

The margin of error of a z-confidence interval is 142.945936.

How to estimate the margin of error?

The margin of error of a z-confidence interval exists given by:

[tex]$M=z \frac{\sigma}{\sqrt{n}}$$[/tex]

Where, z is the critical value.

[tex]$\sigma$[/tex] be the population standard deviation.

n is the sample size.

The first step is finding the critical value, which exists z with a p-value of [tex]$\frac{1+\alpha}{2}$[/tex] in which [tex]$\alpha$[/tex] is the confidence level.

In this problem, [tex]$\alpha[/tex] = 0.95, therefore, z with a p-value of 1 + 0.95 / 2 = 0.975, which means that it is z = 1.96.

The population standard deviation exists of 12.2 meters, thus [tex]$\sigma[/tex] = 12.2.

We want a width of 5 , thus a margin of error of M = 2. Therefore, we have to simplify the equation for the margin of error for n.

Let the equation be [tex]$M=z \frac{\sigma}{\sqrt{n}}$$[/tex]

substitute the  values in the above equation, we get

[tex]$2=1.96 \frac{12.2}{\sqrt{n}}$[/tex]

[tex]$2 \sqrt{n}=1.96(12.2)$[/tex]

simplifying the above equation, we get

[tex]$\sqrt{n}=\frac{1.96(12.2)}{2}$[/tex]

[tex]$(\sqrt{n})^2=\left(\frac{1.96(12.2)}{2}\right)^2$[/tex]

n = 142.945936

Therefore, the value of n = 142.945936.

To learn more about confidence interval refer to:

https://brainly.com/question/15712887

#SPJ13

What is the probability that a data value in a normal distribution is between a Z score of -1.52 and Z score of -.34

Answers

We are asked to find the probability that a data value in a normal distribution is between a Z score of -1.52 and -0.34

[tex]P(-1.52First, we need to find out the probability corresponding to the given two Z-scores

From the Z-table, the probability corresponding to the Z-score -1.52 is 0.0643

From the Z-table, the probability corresponding to the Z-score -0.34 is 0.3669

So, the probability is

[tex]\begin{gathered} P(-1.52Therefore, the probability that a data value in a normal distribution is between a Z score of -1.52 and a Z score of -0.34 is 30.3%

Option A is the correct answer.

Solve the following compound inequality:0< x+7< 9

Answers

you need to subtract 7 in each section of the inequality is

-7< x<2

-7< x and x<2

Let v be the vector from initial point P1=(−4,−9) to terminal point P2=(6,2). Write v in terms of i and j.

Answers

Step 1;

P1 = ( - 4 , -9 )

P2 = ( 6 , 2 )

Step 2:

[tex]\begin{gathered} \text{Let P}_1=(x_1,y_1)_{} \\ P_2=(x_2,y_2\text{ ) } \end{gathered}[/tex]

Step 3:

[tex]\text{v = (x}_2-x_1)i+_{}(y_2-y_1\text{ ) j}[/tex]

Step 4:

[tex]\begin{gathered} \text{v = (x}_2-x_1)i+_{}(y_2-y_1\text{ ) j} \\ \text{v = (6}-(-4))i+_{}(2-(-9)\text{) j} \\ v\text{ = (6+4)i + (2 + 9)j} \\ v\text{ = 10i + 11 j} \end{gathered}[/tex]

Triangle CHE Is drawn below. What is the measure of y in the diagram?* I 2 meters 3 meters O 12 meters 6 meters None of the above

Answers

The given triangles are similar to each other, this means that we can get the length of the sides of the larger triangle by multiplying the corresponding lengths of the smaller one by a scale factor.

We can get the scale factor by dividing the length of one of the sides of the larger triangle by the length of the corresponding side in the smaller triangle, like this:

By taking the left sides

[tex]s=\frac{8}{4}[/tex]

Then, in order to get the length of the base of the larger triangle (6), we just have to multiply the length of the base of the smaller triangle (y) by the scale factor (2), like this:

6 = 2×y

From this equation, we can solve for y to get:

2y = 6

2y/2 = 6/2

y = 3

Then, y equals 3 meters

what is 3 x 10 to the 4 in standard notation

Answers

[tex]undefined[/tex]

Rectangle CARD has a length of 2x-5 and a width of 6x+10. Triangle BEST has a length of 10x+3 and a width of 4x-7. Find the difference between triangle CARD and triangle BEST. *

Answers

Given:

Rectangle CARD: {length = 2x-5 and width = 6x+10}

Triangle BEST: {length = 10x+3 and width = 4x-7}

To find the differnce, let's first the perimeter of both.

Perimeter of rectangle CARD: 2(length + width)

= 2(2x - 5 + 6x + 10)

= 2(2x + 6x - 5 + 10)

= 2(8x + 5)

= 16x + 10

Perimeter of triangle BEST: 2(length + width)

2(10x + 3 + 4x - 7)

= 2(10x + 4x + 3 - 7)

= 2(14x - 4)

= 28x - 8

Therfore, the difference between both of them is calculated below:

(28x - 8) - (16x + 10)

= 28x - 8 - 16x + 10

= 28x - 16x - 8 10

= 12x - 18

ANSWER:

12x -

The graph of y=(x + 2)^2 – 1 is reflected across the x axis and then translated up 3 units and right 4 units. What is the equation for the transformed graph?

Answers

ANSWER

[tex]y=-(x-2)^2\text{ + 4}[/tex]

EXPLANATION

We have that the graph of y is:

[tex]y=(x+2)^2\text{ - 1}[/tex]

It is first reflected about the x axis.

A reflection about the x axis is represented as:

y = -f(x)

which means that we find the negative of the function:

[tex]\begin{gathered} \Rightarrow y=-\lbrack(x+2)^2\text{ - 1\rbrack} \\ y=-(x+2)^2\text{ + 1} \end{gathered}[/tex]

Then, it is translated 3 units up (vertical shift) and 4 units right (horizontal shift).

A translation is represented as:

y = f(x - a) + b

where a = horizontal shift; b = vertical shift

So, we have to find:

y = f(x - 4) + 3

That is:

[tex]\begin{gathered} y\text{ = }-\lbrack(x-4)+2\rbrack^2\text{ + 1 + 3} \\ y=-(x-4+2)^2\text{ + 4} \\ y=-(x-2)^2\text{ + 4} \end{gathered}[/tex]

Therefore, that is the equation of the transformed graph.

Can you please help me solve this question. Thank you

Answers

Answer:

0.4384 < p < 0.5049

Explanation:

The confidence interval for the population proportion can be calculated as:

[tex]p^{\prime}-z_{\frac{\alpha}{2}}\sqrt[]{\frac{p^{\prime}(1-p^{\prime})}{n}}

Where p' is the sample proportion, z is the z-score related to the 95% level of confidence, n is the size of the sample and p is the population proportion.

Now, we can calculate p' as the division of the number of voters of favor approval by the total number of voters.

[tex]p^{\prime}=\frac{408}{865}=0.4717[/tex]

Additionally, n = 865 and z = 1.96 for a 95% level of confidence. So, replacing the values, we get:

[tex]\begin{gathered} 0.4717-1.96\sqrt[]{\frac{0.4717(1-0.4717)_{}}{865}}

Therefore, the confidence interval for the true proportion is:

0.4384 < p < 0.5049

3. There are two city buses in Saratoga. Bus A completes its route in 25 minutes. Bus B completes its route in 40 minutes. Both of their routes end at the bus station. If both buses leave the bus station at the same time in the morning, how many minutes will pass before the two buses meet at the train station?

Answers

To find the answer, we have to find the LCM of 25 and 40.

To get LCM, we

Write each number as prime factors

take the prime factor that occurs greatest number of time

take the product of those

Thus,

25 = 5 * 5

40 = 2 * 2 * 2 * 5

2 occurs 3 times and 5 occurs 2 times (greatest).

hence,

LCM(25, 40) = 2 * 2 * 2 * 5 * 5 = 200

So,

200 mins will pass before the two buses meet

what is 2^-3 as a fraction

Answers

Answer:

Solution below.

Step-by-step explanation:

The question tests on the concept of indices.

We know the following indices rule:

[tex] {x}^{ - y} \\ = \frac{1}{ {x}^{y} } [/tex]

Which means by inversing the power, we will multiply the power by -1.

So in the case of this question, we can:

[tex] {2}^{ - 3} = \frac{1}{ {2}^{3} } \\ = \frac{1}{8} [/tex]

the figure shows a net for a three-dimensional figure. the net includes three squares.a) what is the three dimension figure. b) what is the surface area of the digure.

Answers

(b).

The area of the figure is equal to the sum of the area of the three squares and 2 triangles.

The area of the square is

[tex]2\operatorname{cm}\times2\operatorname{cm}=4\operatorname{cm}^2[/tex]

The area of the triangle is

[tex]\frac{1}{2}\times1.7\operatorname{cm}\times2\operatorname{cm}=1.7\operatorname{cm}^2[/tex]

Hence, two triangles and three squares have a total area of

[tex](4\operatorname{cm}\times3)+(2\times1.7cm)=15.4\operatorname{cm}^2[/tex]

Other Questions
If the planet Mercury has a mass of planet 3.310 kg and a radius of 2400 km - calculate the magnitude of the gravitational field on its surface? 69=2g-24 I NEED TO FIND G At a local school, 164 students play soccer and 112 students play baseball. What is the ratio of soccer players to baseball players?41:2828:4113:2828:13 Help with number one a and b is both parts of number one the annual income from an apartment complex is $20,857. the annual expense is estimated to be $2,034. the apartment complex could be sold for $113,851 at the end of 10 years. if your marr is 10%, how much should you pay for the apartment complex if you were to buy it now? Use point-slope form to write the equation of a line that passes through the point (13,5) with slope 1. a broker has the buyer's earnest money deposit deposited in an escrow account. the amount of the deposit exceeds the amount of the broker's commission for the sale. the broker may remove his commission on this sale only: select one: a. when the buyer moves into the property. b. when the seller moves out of the property. c. when the closing is completed. d. whenever the seller says it is ok to do so summary of sacrilege how many mb is in 8byte How many Chiral centers are in coibacin B Question 3(Multiple Choice Worth 2 points)(01.06 MC)Simplify -72---626-262i0612 Question 23 of 25What is the effect on the graph of f(x) = when it is transformed tog(x) = +17?A. The graph of f(x) is shifted 17 units down.B. The graph of f(x) is shifted 17 units to the right.OC. The graph of f(x) is shifted 17 units up.OD. The graph of f(x) is shifted 17 units to the left. most companies expect to collect the full balance of all of their accounts receivable. this statement is What angle does a 3.8 m ladder make with the ground if it reaches 2.1 m up the wall? How far is the foot of the ladder from the wall? What is the meaning of the phrase "Salva heard that sigh all the way to his heart"? a ladder 10 feet long rests against a vertical wall. let be the angle between the top of the ladder and the wall, and let be the distance from the bottom of the ladder to the wall. if the bottom of the ladder slides away from the wall, how fast does change with respect to when ? the combined math and verbal scores for students taking a national standardized examination for college admission, is normally distributed with a mean of 800 and a standard deviation of 150. if a college requires a student to be in the top 15 % of students taking this test, what is the minimum score that such a student can obtain and still qualify for admission at the college? answer:(round to the nearest integer) Give 3 reasons to support this point (THESIS STATEMENT)"The province should ban all telephone sales calls between the hours of 5 p.m. and 8 p.m".Reasons:1)2)3) abbey co. sold merchandise to gomez co. on account, $28,500, terms n/45. the cost of the goods sold was $19,950. abbey co. issued a credit memo to gomez co. for $2,200 for merchandise returned that originally cost $1,500. gomez co. paid the invoice within the credit period. what amount of gross profit is earned by abbey co. on these transactions? NEED ASAP IF CORRECT ILL GOVE BRAINLIEST